Difference between revisions of "2022 AMC 10A Problems/Problem 3"

m (Solution)
 
(7 intermediate revisions by 5 users not shown)
Line 14: Line 14:
  
 
== Solution 2 ==
 
== Solution 2 ==
Solve this using a system of equations. Let <math>x</math>, <math>y</math>, and <math>z</math> be the three numbers, respectively. We get three equations:
+
Solve this using a system of equations. Let <math>x,y,</math> and <math>z</math> be the three numbers, respectively. We get three equations:
<cmath>x+y+z=96</cmath>
+
<cmath>\begin{align*}
<cmath>x=6z</cmath>
+
x+y+z&=96, \\
<cmath>z=y-40</cmath>
+
x&=6z, \\
Rewriting the third equation gives us <math>y=z+40</math>, so we can substitute <math>x</math> as <math>6z</math> and <math>y</math> as <math>z+40</math>.
+
z&=y-40.
 +
\end{align*}</cmath>
 +
Rewriting the third equation gives us <math>y=z+40,</math> so we can substitute <math>x</math> as <math>6z</math> and <math>y</math> as <math>z+40.</math>
  
 
Therefore, we get
 
Therefore, we get
<cmath>6z+(z+40)+z=96</cmath>
+
<cmath>\begin{align*}
<cmath>8z+40=96</cmath>
+
6z+(z+40)+z&=96 \\
<cmath>8z=56</cmath>
+
8z+40&=96 \\
<cmath>z=7</cmath>
+
8z&=56 \\
 +
z&=7.
 +
\end{align*}</cmath>
 +
Substituting 7 in for <math>z</math> gives us <math>x=6z=6(7)=42</math> and <math>y=z+40=7+40=47.</math>
  
Substituting 7 in for <math>z</math> gives us
+
So, the answer is <math>|x-y|=|42-47|=\boxed{\textbf{(E) } 5}.</math>
<cmath>x=6z=6(7)=42</cmath> and
 
<cmath>y=z+40=7+40=47</cmath>
 
  
So <math>|x-y|=|42-47|=\boxed{\textbf{(E) } 5}</math>
+
~alexdapog A-A
  
~alexdapog A-A
+
== Solution 3 ==
 +
In accordance with Solution 2, <cmath>y = z+40, x = 6z \implies |x-y| = |6z - z - 40| = 5|z - 8| \implies \boxed{\textbf{(E) } 5}.</cmath>
 +
'''vladimir.shelomovskii@gmail.com, vvsss'''
  
 
==Video Solution 1 (Quick and Easy)==
 
==Video Solution 1 (Quick and Easy)==
Line 43: Line 48:
  
 
~Charles3829
 
~Charles3829
 +
 +
==Video Solution 3 (2 minutes) ==
 +
https://youtu.be/7yAh4MtJ8a8?si=Xpc2h85yyqEMOnVb&t=322
 +
 +
~Math-x
 +
 +
==Video Solution 4==
 +
https://youtu.be/BmWgwtKJExw
  
 
== See Also ==
 
== See Also ==

Latest revision as of 11:35, 24 March 2024

Problem

The sum of three numbers is $96.$ The first number is $6$ times the third number, and the third number is $40$ less than the second number. What is the absolute value of the difference between the first and second numbers?

$\textbf{(A) } 1 \qquad \textbf{(B) } 2 \qquad \textbf{(C) } 3 \qquad \textbf{(D) } 4 \qquad \textbf{(E) } 5$

Solution 1

Let $x$ be the third number. It follows that the first number is $6x,$ and the second number is $x+40.$

We have \[6x+(x+40)+x=8x+40=96,\] from which $x=7.$

Therefore, the first number is $42,$ and the second number is $47.$ Their absolute value of the difference is $|42-47|=\boxed{\textbf{(E) } 5}.$

~MRENTHUSIASM

Solution 2

Solve this using a system of equations. Let $x,y,$ and $z$ be the three numbers, respectively. We get three equations: \begin{align*} x+y+z&=96, \\ x&=6z, \\ z&=y-40. \end{align*} Rewriting the third equation gives us $y=z+40,$ so we can substitute $x$ as $6z$ and $y$ as $z+40.$

Therefore, we get \begin{align*} 6z+(z+40)+z&=96 \\ 8z+40&=96 \\ 8z&=56 \\ z&=7. \end{align*} Substituting 7 in for $z$ gives us $x=6z=6(7)=42$ and $y=z+40=7+40=47.$

So, the answer is $|x-y|=|42-47|=\boxed{\textbf{(E) } 5}.$

~alexdapog A-A

Solution 3

In accordance with Solution 2, \[y = z+40, x = 6z \implies |x-y| = |6z - z - 40| = 5|z - 8| \implies \boxed{\textbf{(E) } 5}.\] vladimir.shelomovskii@gmail.com, vvsss

Video Solution 1 (Quick and Easy)

https://youtu.be/v2eJtm4EUkI

~Education, the Study of Everything

Video Solution 2

https://youtu.be/B-5zSnDFVXs

~Charles3829

Video Solution 3 (2 minutes)

https://youtu.be/7yAh4MtJ8a8?si=Xpc2h85yyqEMOnVb&t=322

~Math-x

Video Solution 4

https://youtu.be/BmWgwtKJExw

See Also

2022 AMC 10A (ProblemsAnswer KeyResources)
Preceded by
Problem 2
Followed by
Problem 4
1 2 3 4 5 6 7 8 9 10 11 12 13 14 15 16 17 18 19 20 21 22 23 24 25
All AMC 10 Problems and Solutions
2022 AMC 12A (ProblemsAnswer KeyResources)
Preceded by
Problem 1
Followed by
Problem 3
1 2 3 4 5 6 7 8 9 10 11 12 13 14 15 16 17 18 19 20 21 22 23 24 25
All AMC 12 Problems and Solutions

The problems on this page are copyrighted by the Mathematical Association of America's American Mathematics Competitions. AMC logo.png